Mathcenter Forum  

Go Back   Mathcenter Forum > คณิตศาสตร์โอลิมปิก และอุดมศึกษา > ข้อสอบโอลิมปิก
สมัครสมาชิก คู่มือการใช้ รายชื่อสมาชิก ปฏิทิน ข้อความวันนี้

ตั้งหัวข้อใหม่ Reply
 
เครื่องมือของหัวข้อ ค้นหาในหัวข้อนี้
  #1  
Old 22 เมษายน 2005, 10:44
gools's Avatar
gools gools ไม่อยู่ในระบบ
บัณฑิตฟ้า
 
วันที่สมัครสมาชิก: 26 เมษายน 2004
ข้อความ: 390
gools is on a distinguished road
Post USAMO 2005

1.Determine all composite positive integers \(n\) for which it is
possible to arrange all divisors of \(n\) that are greater than 1 in
a circle so that no two adjacent divisors are relatively prime.

2.Prove that the
system
\[\begin{array}{rcl}
x^6+x^3+x^3y+y & = 147^{157} \\
x^3+x^3y+y^2+y+z^9 & = 157^{147}
\end{array}
\]
has no solutions in integers \(x\), \(y\), and \(z\).

3. Let \(ABC\) be an acute-angled triangle, and let \(P\) and \(Q\) be two
points on side \(BC\). Construct point \(C_1\) in such a way that
convex quadrilateral \(APBC_1\) is cyclic, \(QC_1
\parallel CA\), and \(C_1\) and \(Q\) lie on opposite sides of line
\(AB\). Construct point \(B_1\) in such a way that convex
quadrilateral \(APCB_1\) is cyclic, \(QB_1 \parallel BA\), and \(B_1\)
and \(Q\) lie on opposite sides of line \(AC\). Prove that points
\(B_1, C_1,P\), and \(Q\) lie on a circle.

22 เมษายน 2005 10:46 : ข้อความนี้ถูกแก้ไขแล้ว 1 ครั้ง, ครั้งล่าสุดโดยคุณ gools
ตอบพร้อมอ้างอิงข้อความนี้
  #2  
Old 23 เมษายน 2005, 04:09
nongtum's Avatar
nongtum nongtum ไม่อยู่ในระบบ
ผู้พิทักษ์กฎทั่วไป
 
วันที่สมัครสมาชิก: 10 เมษายน 2005
ข้อความ: 3,246
nongtum is on a distinguished road
Post

ข้อสองครับ วิธีทำอาจดูไม่งามนัก หากคิดผิดตรงไหนช่วยบอกนะครับ

2. We assume first that there are integers x,y,z which satisfy following equations system:
(eq1) \(x^6+x^3+x^{3}y+y=(x^{3}+1)(x^{3}+y)=147^{157}=(7^{2}.3)^{157}\)
(eq2) \(x^3+x^{3}y+y^2+y+z^9=(y+1)(x^{3}+y)+z^9=157^{147}\).

Since the product of two odd numbers are odd, we have \(x^{3}+1\) and \(x^{3}+y\) as odd numbers,
that is \(x^3\) is even and, from our assumption, x is even. Also y and z are odd.

We use then the fact that \(x^3\equiv0\pmod4\), \(y\equiv\pm1\pmod4\) and \(z\equiv\pm1\pmod4\) to verify each case as follows
(following calculations are in mod 4):

equation 2:
$z\equiv1\pmod4$, $y\equiv1\pmod4$ yields $2\cdot1+1\equiv1$ (wrong)
$z\equiv1\pmod4$, $y\equiv-1\pmod4$ yields $(-1)\cdot0+1\equiv1$ (right)
$z\equiv-1\pmod4$, $y\equiv1\pmod4$ yields $2\cdot1-1\equiv1$ (right)
$z\equiv-1\pmod4$, $y\equiv-1\pmod4$ yields $2\cdot0-1\equiv1$ (wrong),

then set the right 'answer sets' in equation 1:
\(1\cdot(\pm1)\equiv(-1)^{157}\equiv-1\),
which give us \(y\equiv-1\pmod4\).

From equation 1 we can also assume that \(x^3+1={7^{\alpha_{1}}.3^{\beta_{1}}}\) and \(x^3+y={7^{\alpha_{2}}.3^{\beta_{2}}}\), where all exponents are integers.

We calculate each factor again in mod 4, which yields:
\(x^3+1\equiv(-1)\cdot{\alpha_{1}}+(-1).{\beta_{1}}=-{\alpha_{1}}-{\beta_{1}}\equiv1\),
and \(x^3+y\equiv\ (-1)\cdot{\alpha_{2}}+(-1).{\beta_{2}}=-{\alpha_{2}}-{\beta_{2}}\equiv-1\).
That means \({\alpha_{1}}+{\beta_{1}}+{\alpha_{2}}+{\beta_{2}}\equiv0\ne-1=157\cdot3\pmod4\),
which yield the desired contradiction, hence complete the proof. (๕๕๕ ซะเมื่อไหร่ )

-------------------------------------
Phew... คงมีวิธีที่ง่ายกว่านี้นะ เพราะอาจคำนวณเศษผิดได้ง่ายมากๆ
Edit: มาแก้ TeX syntax ครับ กระทู้นี้ตอนผมเข้ามาที่นี่ใหม่ๆเลยนะนั่น
__________________
คนไทยร่วมใจอย่าใช้ภาษาวิบัติ
ฝึกพิมพ์สัญลักษณ์สักนิด ชีวิต(คนตอบและคนถาม)จะง่ายขึ้นเยอะ (จริงๆนะ)

Stay Hungry. Stay Foolish.

16 มีนาคม 2007 20:54 : ข้อความนี้ถูกแก้ไขแล้ว 1 ครั้ง, ครั้งล่าสุดโดยคุณ nongtum
ตอบพร้อมอ้างอิงข้อความนี้
  #3  
Old 23 เมษายน 2005, 19:38
warut warut ไม่อยู่ในระบบ
กระบี่ไร้สภาพ
 
วันที่สมัครสมาชิก: 24 พฤศจิกายน 2001
ข้อความ: 1,627
warut is on a distinguished road
Post

ผมก็สนใจข้อ 2 เหมือนกันครับ แต่พยายามคิดเท่าไหร่ก็ไม่ออกสักที ก็พอดีคุณ nongtum
มาตอบ แต่ไม่รู้เป็นไงมีตัวแปลกๆปนอยู่เต็มไปหมด อ่านยากมากเลยครับ แต่ผมก็พยายาม
ติดตามวิธีพิสูจน์ไปเรื่อยๆ ก็เข้าใจและเห็นด้วยมาตลอด แต่ตอนสุดท้ายนี่สิครับผมไม่แน่ใจ
ว่าคุณ nongtum เขียนอย่างนี้รึเปล่า\[x^3+1\equiv
(-1)\cdot\alpha_1+(-1)\cdot\beta_1\pmod4\]ถ้าใช่ผมว่าตรงนี้ไม่น่าจะถูกนะครับ จาก \(x^3+1=7^{\alpha_1}\cdot3^{\beta_1}\) เราควรจะได้ว่า\[x^3+1\equiv
(-1)^{\alpha_1+\beta_1}\pmod4\]มากกว่านะครับ ซึ่งถ้าที่ผมคิดต่อไปไม่ผิด นี่จะยังไม่เพียงพอที่จะพิสูจน์โจทย์ข้อนี้ได้ครับ
ตอบพร้อมอ้างอิงข้อความนี้
  #4  
Old 24 เมษายน 2005, 02:23
gon's Avatar
gon gon ไม่อยู่ในระบบ
ผู้พิทักษ์กฎขั้นสูง
 
วันที่สมัครสมาชิก: 29 มีนาคม 2001
ข้อความ: 4,608
gon is on a distinguished road
Smile

จะเล่น Usamo กันแล้วหรือครับ. ดูผ่าน ๆ โจทย์ข้อ 2. ดูแล้วเข้าใจง่ายสุดเลยนะครับ. ไม่ปวดหัวดี แต่ไม่รู้ถ้าคิดแล้วจะปวดหัวหรือเปล่า เดี๋ยวว่าง ๆ ผม จะลองคิดบ้างครับ.
ตอบพร้อมอ้างอิงข้อความนี้
  #5  
Old 24 เมษายน 2005, 15:47
nongtum's Avatar
nongtum nongtum ไม่อยู่ในระบบ
ผู้พิทักษ์กฎทั่วไป
 
วันที่สมัครสมาชิก: 10 เมษายน 2005
ข้อความ: 3,246
nongtum is on a distinguished road
Post

อ้างอิง:
ข้อความเดิมของคุณ warut:
ผมก็สนใจข้อ 2 เหมือนกันครับ แต่พยายามคิดเท่าไหร่ก็ไม่ออกสักที ก็พอดีคุณ nongtum
มาตอบ แต่ไม่รู้เป็นไงมีตัวแปลกๆปนอยู่เต็มไปหมด อ่านยากมากเลยครับ แต่ผมก็พยายาม
ติดตามวิธีพิสูจน์ไปเรื่อยๆ ก็เข้าใจและเห็นด้วยมาตลอด แต่ตอนสุดท้ายนี่สิครับผมไม่แน่ใจ
ว่าคุณ nongtum เขียนอย่างนี้รึเปล่า\[x^3+1\equiv
(-1)\cdot\alpha_1+(-1)\cdot\beta_1\pmod4\]ถ้าใช่ผมว่าตรงนี้ไม่น่าจะถูกนะครับ จาก \(x^3+1=7^{\alpha_1}\cdot3^{\beta_1}\) เราควรจะได้ว่า\[x^3+1\equiv
(-1)^{\alpha_1+\beta_1}\pmod4\]มากกว่านะครับ ซึ่งถ้าที่ผมคิดต่อไปไม่ผิด นี่จะยังไม่เพียงพอที่จะพิสูจน์โจทย์ข้อนี้ได้ครับ
ขอบคุณมากครับสำหรับตำท้วงติง ตอนนี้เลยยังคิดไม่ออกเหมือนกัน คือคิดไปคิดมาทำท่าว่าจะไม่จบแค่ mod 8 เอาซะอย่างงั้น แต่ยังเชื่อว่าสามารถแก้ไดด้วยวิธีที่พิมพ์ให้ดู เอาเป็นว่าผมจะมาแก้อีกทีตอน'คิดเอง'ออกละกันครับ ขอหลบไปปั่นการบ้านที่กำลังท่วมหัวก่อน
________________________________
I love spoilers.
__________________
คนไทยร่วมใจอย่าใช้ภาษาวิบัติ
ฝึกพิมพ์สัญลักษณ์สักนิด ชีวิต(คนตอบและคนถาม)จะง่ายขึ้นเยอะ (จริงๆนะ)

Stay Hungry. Stay Foolish.
ตอบพร้อมอ้างอิงข้อความนี้
  #6  
Old 16 มีนาคม 2007, 19:45
warut warut ไม่อยู่ในระบบ
กระบี่ไร้สภาพ
 
วันที่สมัครสมาชิก: 24 พฤศจิกายน 2001
ข้อความ: 1,627
warut is on a distinguished road
Smile

อ้างอิง:
ข้อความเดิมของคุณ gools:
2.Prove that the system
\[\begin{array}{rcl}
x^6+x^3+x^3y+y & = & 147^{157} \\
x^3+x^3y+y^2+y+z^9 & = & 157^{147}
\end{array}
\]
has no solutions in integers \(x\), \(y\), and \(z\).
ผมลองทำตาม hint ที่คุณ nongtum ให้มาคือใช้ modulo 13 ดังนั้นสมภาคทั้งหมดข้างล่างนี้เป็น mod 13 นะครับ นั่นทำให้สมการทั้งสองกลายเป็น $$\begin{array}{lrcl} (1) & x^6+x^3+x^3y+y & \equiv & 4 \\ (2) & x^3+x^3y+y^2+y+z^9 & \equiv & 1 \end{array}$$ เนื่องจาก $x^3\equiv 0, \pm1, \pm5$ และ $x^6\equiv 0, \pm1$ ดังนั้น $z^9= z^6z^3 \equiv 0, \pm1, \pm5$

กรณีที่ 1: $x^3\equiv0$
จาก $(1)$ เราจะได้ $y\equiv4$ แทนค่าลงใน $(2)$ จะได้ $z^9\equiv-6$ ดังนั้นจึงไม่มีคำตอบในกรณีนี้

กรณีที่ 2: $x^3\equiv1$
จาก $(1)$ เราจะได้ $y\equiv1$ แทนค่าลงใน $(2)$ จะได้ $z^9\equiv-3$ ดังนั้นจึงไม่มีคำตอบในกรณีนี้

กรณีที่ 3: $x^3\equiv-1$
เราจะพบว่าไม่มี $y$ ที่สอดคล้องกับ $(1)$ จึงไม่มีคำตอบในกรณีนี้

กรณีที่ 4: $x^3\equiv5$
จาก $(1)$ เราจะได้ $y\equiv0$ แทนค่าลงใน $(2)$ จะได้ $z^9\equiv-4$ ดังนั้นจึงไม่มีคำตอบในกรณีนี้

กรณีที่ 5: $x^3\equiv-5$
จาก $(1)$ เราจะได้ $y\equiv4$ แทนค่าลงใน $(2)$ จะได้ $z^9\equiv6$ ดังนั้นจึงไม่มีคำตอบในกรณีนี้เช่นกัน

เราจึงสรุปได้ว่า ระบบสมการของโจทย์ข้อนี้ ไม่มีคำตอบเป็นจำนวนเต็มครับ

ป.ล. ผมไม่ได้ลอง search หาเฉลยของข้อนี้นะครับ แต่คิดว่าน่าจะคล้ายกับที่ผมทำ เพราะส่วนที่ยากที่สุดน่าจะเป็นการหา modulus ที่เหมาะสม ซึ่งผมลองให้คอมพ์หา prime moduli ที่ใช้ได้ ก็เจอเพียง 13 กับ 19 ตามที่คุณ nongtum บอกไว้ใน hint นั่นแหละครับ
ตอบพร้อมอ้างอิงข้อความนี้
ตั้งหัวข้อใหม่ Reply


หัวข้อคล้ายคลึงกัน
หัวข้อ ผู้ตั้งหัวข้อ ห้อง คำตอบ ข้อความล่าสุด
ผลการแข่งขัน IMO 2005 gon ปัญหาคณิตศาสตร์ทั่วไป 7 22 กรกฎาคม 2005 14:21
มาแล้ว ๆ IMO 2005 gon ข้อสอบโอลิมปิก 10 21 กรกฎาคม 2005 21:16
Vietnam Mathematical Olympiad 2005 problem 4 gools ข้อสอบโอลิมปิก 8 18 มิถุนายน 2005 21:09
APMO 2005 #4 devil jr. คณิตศาสตร์อุดมศึกษา 0 09 พฤษภาคม 2005 22:12
APMO 2005 aaaa อสมการ 21 30 มีนาคม 2005 22:52


กฎการส่งข้อความ
คุณ ไม่สามารถ ตั้งหัวข้อใหม่ได้
คุณ ไม่สามารถ ตอบหัวข้อได้
คุณ ไม่สามารถ แนบไฟล์และเอกสารได้
คุณ ไม่สามารถ แก้ไขข้อความของคุณเองได้

vB code is On
Smilies are On
[IMG] code is On
HTML code is Off
ทางลัดสู่ห้อง


เวลาที่แสดงทั้งหมด เป็นเวลาที่ประเทศไทย (GMT +7) ขณะนี้เป็นเวลา 02:58


Powered by vBulletin® Copyright ©2000 - 2024, Jelsoft Enterprises Ltd.
Modified by Jetsada Karnpracha